rozwiązanie zadania 4.3

This commit is contained in:
Kacper Donat 2018-02-25 17:37:30 +01:00
parent 3ec86d1de8
commit ac0fb9a145
3 changed files with 313 additions and 277 deletions

View File

@ -24,6 +24,12 @@
\let\oldthetask\thetask
\let\oldthesubtask\thesubtask
\renewcommand\leq\leqslant
\renewcommand\geq\geqslant
\let\oldemptyset\emptyset
\renewcommand\emptyset\varnothing
\renewcommand{\thesection}{\arabic{section}}
\renewcommand{\thesubtask}{\alph{subtask}}
\renewcommand{\thetask}{\thesection.\oldthetask}
@ -60,6 +66,7 @@
\newcommand{\NP}{\texttt{NP}\xspace}
\newcommand{\NPC}{\texttt{NPC}\xspace}
\newcommand{\NPH}{\texttt{NPH}\xspace}
\newcommand{\NPI}{\texttt{NPI}\xspace}
\renewcommand{\P}{\texttt{P}\xspace}
@ -86,6 +93,3 @@
\floatstyle{plain}
\newfloat{program}{thp}{lop}
\floatname{program}{Program}
\renewcommand\leq\leqslant
\renewcommand\geq\geqslant

View File

@ -6,28 +6,28 @@
\subtask{Jeśli nie potrafisz, to spróbuj udowodnić jego \NP-zupełność}
\begin{solution}
Przy rozwiązywaniu tego zadania, jedną z pierwszych myśli może być, że jest to problem zbliżony
do problemu sumy podzbioru - i jest to prawdą, jest to specyficzny wypadek problemu sumy podzbioru.
Przy rozwiązywaniu tego zadania, jedną z pierwszych myśli może być, że jest to problem zbliżony
do problemu sumy podzbioru - i jest to prawdą, jest to specyficzny wypadek problemu sumy podzbioru.
Zauważmy jednak, że do dyspozycji mamy tylko i wyłącznie 6 wartości: 1, 2, 5, 10, 20, 50 oraz, że $n$ jest stałe i wynosi $100$.
Łatwo zauważyć, że istnieją bardzo proste przypadki, które rozwiązują nasz problem. Przykładowo, jeżeli mamy 2 bankonoty 50zł to
jesteśmy w stanie zapłacić 100zł bez problemu, tak samo mając 5 banknotów 20zł czy nawet 100 złotówek.
Zauważmy jednak, że do dyspozycji mamy tylko i wyłącznie 6 wartości: 1, 2, 5, 10, 20, 50 oraz, że $n$ jest stałe i wynosi $100$.
Łatwo zauważyć, że istnieją bardzo proste przypadki, które rozwiązują nasz problem. Przykładowo, jeżeli mamy 2 bankonoty 50zł to
jesteśmy w stanie zapłacić 100zł bez problemu, tak samo mając 5 banknotów 20zł czy nawet 100 złotówek.
\begin{table}[H]
\begin{tabular}{rrrrrr}
1zł & 2zł & 5zł & 10zł & 20zł & 50zł \\ \hline
- & - & - & - & - & 2 \\
- & - & - & 1 & 2 & 1 \\
- & - & - & 3 & 1 & 1 \\
- & - & 2 & 2 & 1 & 1 \\
\vdots & \vdots & \vdots & \vdots & \vdots & \vdots
\end{tabular}
\centering
\caption{Możliwe kombinacje tworzące 100zł}
\end{table}
\begin{table}[H]
\begin{tabular}{rrrrrr}
1zł & 2zł & 5zł & 10zł & 20zł & 50zł \\ \hline
- & - & - & - & - & 2 \\
- & - & - & 1 & 2 & 1 \\
- & - & - & 3 & 1 & 1 \\
- & - & 2 & 2 & 1 & 1 \\
\vdots & \vdots & \vdots & \vdots & \vdots & \vdots
\end{tabular}
\centering
\caption{Możliwe kombinacje tworzące 100zł}
\end{table}
Stąd wynika, że istnieje jedynie skończona ilość kombinacji, którymi możemy utworzyć 100zł. Wystarczy zatem sprawdzić
je wszystkie, a to można wykonac zawsze w tym samym czasie - uzyskany algorytm jest $O(1)$.
Stąd wynika, że istnieje jedynie skończona ilość kombinacji, którymi możemy utworzyć 100zł. Wystarczy zatem sprawdzić
je wszystkie, a to można wykonac zawsze w tym samym czasie - uzyskany algorytm jest $O(1)$.
\end{solution}
\task
@ -37,37 +37,37 @@ je wszystkie, a to można wykonac zawsze w tym samym czasie - uzyskany algorytm
byś mógł skorzystać z mojego programu? Zaprojektuj taki program i oszacuj jego złożoność.
\begin{solution}
Nasz graf $G$ może być w zasadzie dowolnym grafem 100-wierzchołkowym, w szczególności nie wiemy nic o tym czy jest
hamiltonowski czy nie. Do dyspozycji mamy program, który jest w stanie odpowiedzieć na nasze pytanie,
ale tylko jeżeli graf \textbf{nie} jest hamiltonowski. Potrzebujemy zatem sposobu na przetworzenie naszego grafu $G$ tak,
aby nie miał szans być hamiltonowski - jednak to przekształcenie nie może wpływać na odpowiedź na nasze pytanie.
Nasz graf $G$ może być w zasadzie dowolnym grafem 100-wierzchołkowym, w szczególności nie wiemy nic o tym czy jest
hamiltonowski czy nie. Do dyspozycji mamy program, który jest w stanie odpowiedzieć na nasze pytanie,
ale tylko jeżeli graf \textbf{nie} jest hamiltonowski. Potrzebujemy zatem sposobu na przetworzenie naszego grafu $G$ tak,
aby nie miał szans być hamiltonowski - jednak to przekształcenie nie może wpływać na odpowiedź na nasze pytanie.
Na wstępie przypomnijmy, że graf hamiltonowski to taki, w którym istnieje cykl pozwalający przejść po
wszystkich wierzchołkach dokładnie raz. Co znaczy, że każdy graf hamiltonowski musi być przede wszystkim grafem spójnym.
Co za tym idzie, jeżeli przekształcimy nasz graf G w taki sposób, że nie będzie spójny to będziemy mieli pewność,
że nie będzie hamiltonowski.
Na wstępie przypomnijmy, że graf hamiltonowski to taki, w którym istnieje cykl pozwalający przejść po
wszystkich wierzchołkach dokładnie raz. Co znaczy, że każdy graf hamiltonowski musi być przede wszystkim grafem spójnym.
Co za tym idzie, jeżeli przekształcimy nasz graf G w taki sposób, że nie będzie spójny to będziemy mieli pewność,
że nie będzie hamiltonowski.
Nie możemy usuwać ani dodawać krawędzi między wierzchołkami naszego grafu $G$, ponieważ mogłoby to wpłynąć na istnienie
w nim kliki - najprostszym rozwiązaniem będzie więc dodanie zupełnie nowego wierzchołka, nazwijmy go $v_{101}$,
który nie jest połączony z żadnym innym.
Nie możemy usuwać ani dodawać krawędzi między wierzchołkami naszego grafu $G$, ponieważ mogłoby to wpłynąć na istnienie
w nim kliki - najprostszym rozwiązaniem będzie więc dodanie zupełnie nowego wierzchołka, nazwijmy go $v_{101}$,
który nie jest połączony z żadnym innym.
\begin{figure}[H]
\centering
\input{gfx/hamilton.tex}
\caption{Transformacja w graf niehamiltonowski}
\end{figure}
\begin{figure}[H]
\centering
\input{gfx/hamilton.tex}
\caption{Transformacja w graf niehamiltonowski}
\end{figure}
Dodanie jednego swobodnego wierzchołka nie wpłynie na istnienie kliki $K_{10}$ bądź większej ponieważ jeden wierzchołek
nie wystarczy na utworzenie kliki, a nie dodajemy żadnych krawędzi, które mogłyby doprowadzić do tego, że klika nagle powstała.
Dodanie jednego swobodnego wierzchołka nie wpłynie na istnienie kliki $K_{10}$ bądź większej ponieważ jeden wierzchołek
nie wystarczy na utworzenie kliki, a nie dodajemy żadnych krawędzi, które mogłyby doprowadzić do tego, że klika nagle powstała.
Innym możliwym rozwiązaniem jest dodanie wierzchołka połączonego z tylko jednym wierzchołkiem - jedyny sposób aby się
do niego dostać będzie przez 1 wierzchołek, a żeby przejść dalej będziemy musieli się cofnąć do wierzchołka, w którym już byliśmy.
Innym możliwym rozwiązaniem jest dodanie wierzchołka połączonego z tylko jednym wierzchołkiem - jedyny sposób aby się
do niego dostać będzie przez 1 wierzchołek, a żeby przejść dalej będziemy musieli się cofnąć do wierzchołka, w którym już byliśmy.
Jeżeli zaś chodzi o złożoność, to ważnym jest, że graf jest podany w formie macierzy sąsiedztwa, czyli aby dodać
wierzchołek musimy do macierzy dodać 1 wiersz i 1 kolumnę - czyli w zasadzie utworzyć nową macierz. Operacja ta wymaga
utworzenia $(n+1) \times (n+1)$ komórek, czyli teoretycznie jej złożoność to $O(n^2)$ jednak biorąc pod uwagę, że dla
grafu $G$ z definicji $n = 100$ możemy powiedzieć, że operacja ta dla każdego takiego grafu G jest do wykonania w czasie
stałym, czyli $O(1)$ - obie odpowiedzi będą poprawne.
Jeżeli zaś chodzi o złożoność, to ważnym jest, że graf jest podany w formie macierzy sąsiedztwa, czyli aby dodać
wierzchołek musimy do macierzy dodać 1 wiersz i 1 kolumnę - czyli w zasadzie utworzyć nową macierz. Operacja ta wymaga
utworzenia $(n+1) \times (n+1)$ komórek, czyli teoretycznie jej złożoność to $O(n^2)$ jednak biorąc pod uwagę, że dla
grafu $G$ z definicji $n = 100$ możemy powiedzieć, że operacja ta dla każdego takiego grafu G jest do wykonania w czasie
stałym, czyli $O(1)$ - obie odpowiedzi będą poprawne.
\end{solution}
\task{\textit{Ograniczony Problem Kliki} - \problem{OPK} - zdefiniowany jest następująco: ,,Dany jest graf $G$ i liczby
@ -76,38 +76,38 @@ naturalne $a$, $b$ takie, że $a \leq b$, czy w $G$ istnieje klika o rozmiarze $
\subtask{$\problem{OPK} \in \problem{NPC}$}
\begin{solution}
Aby udowodnić, że dany problem należy do klasy \NP należy zrobić jedną z dwóch rzeczy - przedstawić niedeterministyczny
algorytm rozwiązujący ten problem w czasie wielomianowym, bądź zaprezentować wielomianowy algorytm sprawdzający
rozwiązanie tego problemu. Moim zdaniem, podejście drugie jest o wiele łatwiejsze i przyjemniejsze - zatem to właśnie
je zaprezentuję i będę wykorzystywał.
Aby udowodnić, że dany problem należy do klasy \NP należy zrobić jedną z dwóch rzeczy - przedstawić niedeterministyczny
algorytm rozwiązujący ten problem w czasie wielomianowym, bądź zaprezentować wielomianowy algorytm sprawdzający
rozwiązanie tego problemu. Moim zdaniem, podejście drugie jest o wiele łatwiejsze i przyjemniejsze - zatem to właśnie
je zaprezentuję i będę wykorzystywał.
Aby sprawdzić rozwiązanie, potrzebujemy więcej informacji niż proste \textbf{tak} lub \textbf{nie} - taką dodatkową
informację nazywamy \textit{certyfikatem}, przykładowo w wypadku pytania, czy liczba $n$ jest złożona, certyfikatem mogą
być dwie liczby $p$ i $q$ takie, że $n = p \cdot q$ - mając taki certyfikat łatwo jest potwierdzić, że faktycznie
liczba jest złożona.
Aby sprawdzić rozwiązanie, potrzebujemy więcej informacji niż proste \textbf{tak} lub \textbf{nie} - taką dodatkową
informację nazywamy \textit{certyfikatem}, przykładowo w wypadku pytania, czy liczba $n$ jest złożona, certyfikatem mogą
być dwie liczby $p$ i $q$ takie, że $n = p \cdot q$ - mając taki certyfikat łatwo jest potwierdzić, że faktycznie
liczba jest złożona.
W naszym wypadku certyfikatem może być lista wierzchołków tworząca klikę $K_n$. Określenie $r$ będzie wymagało
sprawdzenia długości listy, co z pewnością można wykonać w czasie wielomianowym, a sprawdzenie czy wierzchołki tworzą
klikę będzie wymagało sprawdzenia czy faktycznie każdy wierzchołek sąsiaduje z każdym innym wierzchołkiem z listy - co
też można wykonać w czasie wielomianowym. Stąd wynika, że problem należy do klasy \NP.
W naszym wypadku certyfikatem może być lista wierzchołków tworząca klikę $K_n$. Określenie $r$ będzie wymagało
sprawdzenia długości listy, co z pewnością można wykonać w czasie wielomianowym, a sprawdzenie czy wierzchołki tworzą
klikę będzie wymagało sprawdzenia czy faktycznie każdy wierzchołek sąsiaduje z każdym innym wierzchołkiem z listy - co
też można wykonać w czasie wielomianowym. Stąd wynika, że problem należy do klasy \NP.
Dowód, że problem należy do \NPC jest trochę trudniejszy do przeprowadzenia - należy mianowicie wykonać
$\alpha$-redukcję jakiegoś problemu, który wiemy że jest \NPC do problemu, który badamy. Naturalnym kandydatem tutaj
wydaje się być problem kliki:
``Czy w grafie $G$ istnieje klika wielkości $n$?,,
Dowód, że problem należy do \NPC jest trochę trudniejszy do przeprowadzenia - należy mianowicie wykonać
$\alpha$-redukcję jakiegoś problemu, który wiemy że jest \NPC do problemu, który badamy. Naturalnym kandydatem tutaj
wydaje się być problem kliki:
``Czy w grafie $G$ istnieje klika wielkości $n$?,,
\begin{figure}[H]
\centering
\input{./gfx/klika-opk.tex}
\caption{schemat $\alpha$-redukcji \problem{kliki} do \problem{OPK}}
\end{figure}
\begin{figure}[H]
\centering
\input{./gfx/klika-opk.tex}
\caption{schemat $\alpha$-redukcji \problem{kliki} do \problem{OPK}}
\end{figure}
Aby dokonać $\alpha$-redukcji musimy w wielomianowym czasie na podstawie danych $G, n$ znaleźć takie $G_1, a, b$ aby
rozwiązując problem \problem{OPK} uzyskać odpowiedź na problem kliki. Wiemy, że \problem{OPK} zwróci nam prawdę jeżeli
w grafie $G_1$ będzie klika o rozmiarze $r$ takim, że $a \leq r \leq b$. Jeżeli chcemy znaleźć klikę konkretnego
rozmiaru, wystarczy zauważyć, że jeżeli $a = b \land A \leq r \leq b$ to $a = b = r$. Nasza $\alpha$-redukcja mogłaby
zatem wyglądać następująco: $G_1 = G, a = n, b = n$. Co z pewnością możemy dokonać w czasie wielomianowym. Fakt, że
takie podstawienie zachowuje problem udowodniliśmy już powyżej.
Aby dokonać $\alpha$-redukcji musimy w wielomianowym czasie na podstawie danych $G, n$ znaleźć takie $G_1, a, b$ aby
rozwiązując problem \problem{OPK} uzyskać odpowiedź na problem kliki. Wiemy, że \problem{OPK} zwróci nam prawdę jeżeli
w grafie $G_1$ będzie klika o rozmiarze $r$ takim, że $a \leq r \leq b$. Jeżeli chcemy znaleźć klikę konkretnego
rozmiaru, wystarczy zauważyć, że jeżeli $a = b \land A \leq r \leq b$ to $a = b = r$. Nasza $\alpha$-redukcja mogłaby
zatem wyglądać następująco: $G_1 = G, a = n, b = n$. Co z pewnością możemy dokonać w czasie wielomianowym. Fakt, że
takie podstawienie zachowuje problem udowodniliśmy już powyżej.
\end{solution}
\task{\textit{Problem zanurzenia} (\problem{PZ}) grafu w innym grafie zdefiniowany jest następująco. Dane są dwa grafy:
@ -116,47 +116,47 @@ $f: V(G) \to V(H)$, że każdej krawędzi $\{u, v\} \in G$ odpowiada krawędź $
problemem NP-zupełnym.}
\begin{solution}
Przed dowiedzeniem, że problem jest \NPC najpierw musimy udowodnić, że w ogóle znajduje się w klasie \NP. Po raz
kolejny wykorzystamy tutaj podejście z certyfikatem, w tym wypadku będzie nim funkcja $f$. Nie rozważamy tutaj tego
jak trudno jest wyliczyć na komputerze wynik działania funkcji $f$, zatem zakładamy, że $f = O(1)$. Przetworzenie
wszystkich krawędzi można zatem wykonać w czasie $O(m(G))$. Aby sprawdzić czy krawędź istnieje w grafie $H$ w
najgorszym wypadku musimy przejrzeć wszystkie krawędzie - co daje nam czas $O(m(H))$, ponieważ dla każdej przetworzonej
krawędzi w $G$ musimy sprawdzić jej istnienie w $H$ to w najgorszym wypadku uzyskamy czas
$O(m(H) \cdot m(G))$ - czyli wielomianowy.
Przed dowiedzeniem, że problem jest \NPC najpierw musimy udowodnić, że w ogóle znajduje się w klasie \NP. Po raz
kolejny wykorzystamy tutaj podejście z certyfikatem, w tym wypadku będzie nim funkcja $f$. Nie rozważamy tutaj tego
jak trudno jest wyliczyć na komputerze wynik działania funkcji $f$, zatem zakładamy, że $f = O(1)$. Przetworzenie
wszystkich krawędzi można zatem wykonać w czasie $O(m(G))$. Aby sprawdzić czy krawędź istnieje w grafie $H$ w
najgorszym wypadku musimy przejrzeć wszystkie krawędzie - co daje nam czas $O(m(H))$, ponieważ dla każdej przetworzonej
krawędzi w $G$ musimy sprawdzić jej istnienie w $H$ to w najgorszym wypadku uzyskamy czas
$O(m(H) \cdot m(G))$ - czyli wielomianowy.
Aby udowodnić, że \problem{PZ} jest \NPC musimy znaleźć $\alpha$-redukcje jakiegoś innego, najlepiej zbliżonego
problemu do \problem{PZ}. Na początek zauważmy, że $f$ musi być funkcją różnowartościową, ponieważ gdyby nie była, to
istniałyby takie $u_1 \neq u_2$, że $f(u_1) = f(u_2) = v$, co dawałoby nieistniejącą krawędź $\{ v, v \}$
(tj. wierzchołek byłby połączony z samym sobą). Gdybyśmy natomiast rozpatrywali grafy mogące zawierać takie pętle
własne, to problem dla takich grafów oraz funkcji nieróżnowartościowej stawałby się trywialny - wystarczyłoby zmapować
wszystkie wierzchołki na ten jeden z pętlą własną i taka funkcja istniałaby zawsze.
Aby udowodnić, że \problem{PZ} jest \NPC musimy znaleźć $\alpha$-redukcje jakiegoś innego, najlepiej zbliżonego
problemu do \problem{PZ}. Na początek zauważmy, że $f$ musi być funkcją różnowartościową, ponieważ gdyby nie była, to
istniałyby takie $u_1 \neq u_2$, że $f(u_1) = f(u_2) = v$, co dawałoby nieistniejącą krawędź $\{ v, v \}$
(tj. wierzchołek byłby połączony z samym sobą). Gdybyśmy natomiast rozpatrywali grafy mogące zawierać takie pętle
własne, to problem dla takich grafów oraz funkcji nieróżnowartościowej stawałby się trywialny - wystarczyłoby zmapować
wszystkie wierzchołki na ten jeden z pętlą własną i taka funkcja istniałaby zawsze.
Z tego, że $f$ jest różnowartościowa wynika, że jeżeli wierzchołek w grafie G $\deg u = \delta$ to w grafie H
$\deg f(u) \geq \delta$. Można to zaobserwować na rysunku poniżej.
Z tego, że $f$ jest różnowartościowa wynika, że jeżeli wierzchołek w grafie G $\deg u = \delta$ to w grafie H
$\deg f(u) \geq \delta$. Można to zaobserwować na rysunku poniżej.
\begin{figure}[H]
\centering
\input{gfx/mapowanie.tex}
\caption{Powiązane wierzchołki mają ten sam kolor w obu grafach.}
\end{figure}
\begin{figure}[H]
\centering
\input{gfx/mapowanie.tex}
\caption{Powiązane wierzchołki mają ten sam kolor w obu grafach.}
\end{figure}
Biorąc pod uwagę ten tok rozumowania, możemy zauważyć że problem ten jest nam w stanie łatwo odpowiedzieć czy w danym
grafie $H$ występuje podgraf $G$. Znanym problemem \NPC, który wymaga sprawdzenia obecności danego podgrafu w innym
grafie jest oczywiście problem kliki.
Biorąc pod uwagę ten tok rozumowania, możemy zauważyć że problem ten jest nam w stanie łatwo odpowiedzieć czy w danym
grafie $H$ występuje podgraf $G$. Znanym problemem \NPC, który wymaga sprawdzenia obecności danego podgrafu w innym
grafie jest oczywiście problem kliki.
\begin{figure}[H]
\centering
\input{./gfx/klika-pz.tex}
\caption{schemat $\alpha$-redukcji kliki do PZ}
\end{figure}
\begin{figure}[H]
\centering
\input{./gfx/klika-pz.tex}
\caption{schemat $\alpha$-redukcji kliki do PZ}
\end{figure}
$\alpha$-redukcja polega na sprawdzeniu czy graf pełny $K_n$ jest podgrafem grafu $G_0$, czyli dla problemu \problem{PZ}
otrzymalibyśmy tak skonstruowane dane wejściowe:
$G = K_n, H = G_0$.
$\alpha$-redukcja polega na sprawdzeniu czy graf pełny $K_n$ jest podgrafem grafu $G_0$, czyli dla problemu \problem{PZ}
otrzymalibyśmy tak skonstruowane dane wejściowe:
$G = K_n, H = G_0$.
Problem jest zachowany ponieważ jeżeli w grafie $G_0$ występuje klika o rozmiarze $n$ to możemy każdemu wierzchołkowi
kliki znalezionej w $G_0$ przypisać unikalny wierzchołek w $K_n$. W drugą stronę analogicznie, jeżeli istnieje takie
przypisanie to w grafie $G_0$ musi znajdować się klika $K_n$.
Problem jest zachowany ponieważ jeżeli w grafie $G_0$ występuje klika o rozmiarze $n$ to możemy każdemu wierzchołkowi
kliki znalezionej w $G_0$ przypisać unikalny wierzchołek w $K_n$. W drugą stronę analogicznie, jeżeli istnieje takie
przypisanie to w grafie $G_0$ musi znajdować się klika $K_n$.
\end{solution}
\task{Przedyskutuj złożoność obliczeniową następujących problemów decyzyjnych, zakładając, że $G$ jest kubiczny
@ -180,32 +180,32 @@ brzmi ona \textbf{tak} lub \textbf{nie}.}
$G = C_{2k-1}$ (czyli będącego cyklem nieparzystym i $G = K_n$)}
\begin{solution}
Problem dotyczy liczby chromatycznej grafu - $\chi(G)$ - czyli minimalnej liczby kolorów potrzebnej do poprawnego
pokolorowania grafu. Graf można pokolorować 1 kolorem $\chi(G) = 1$, tylko wtedy kiedy nie ma on krawędzi, a tym wypadku
wiemy, że graf jest 3-regularny czyli dla każdego wierzchołka $v \in G$ wiemy, że jest połączony z 3 innymi
wierzchołkami - stąd możemy wywnioskować, że dla tego typu grafów $\chi(G) \geq 2$.
Problem dotyczy liczby chromatycznej grafu - $\chi(G)$ - czyli minimalnej liczby kolorów potrzebnej do poprawnego
pokolorowania grafu. Graf można pokolorować 1 kolorem $\chi(G) = 1$, tylko wtedy kiedy nie ma on krawędzi, a tym wypadku
wiemy, że graf jest 3-regularny czyli dla każdego wierzchołka $v \in G$ wiemy, że jest połączony z 3 innymi
wierzchołkami - stąd możemy wywnioskować, że dla tego typu grafów $\chi(G) \geq 2$.
Dodatkowo wiemy, że każdy graf dwudzielny można pokolorować za pomocą dokładnie 2 kolorów ponieważ zawiera 2 grupy
niepołączonych ze sobą wierzchołków - a sprawdzenie dwudzielności grafu można osiągnąć za pomocą BFS-a lub DFS-a,
czyli w złożoności $O(m+ n)$.
Dodatkowo wiemy, że każdy graf dwudzielny można pokolorować za pomocą dokładnie 2 kolorów ponieważ zawiera 2 grupy
niepołączonych ze sobą wierzchołków - a sprawdzenie dwudzielności grafu można osiągnąć za pomocą BFS-a lub DFS-a,
czyli w złożoności $O(m+ n)$.
Z wskazówki znamy twierdzenie Brooksa - $\chi(G) \leq \Delta$, w naszym wypadku z 3-regularności oczywistym jest, że
$\Delta = 3$, czyli $\chi(G) \leq 3$, \textbf{CHYBA, że} graf jest nieparzystym cyklem - co możemy wyeliminować ponieważ
cykl nie jest 3-regularny, lub $G = K_4$, gdzyż klika rozmiaru 4 jest 3-regularna (Ogólnie kliki rozmiaru $n$
$(n - 1)$-regularne) i tylko w tym jednym wypadku będziemy potrzebować 4 kolorów, ale nigdy więcej.
Z wskazówki znamy twierdzenie Brooksa - $\chi(G) \leq \Delta$, w naszym wypadku z 3-regularności oczywistym jest, że
$\Delta = 3$, czyli $\chi(G) \leq 3$, \textbf{CHYBA, że} graf jest nieparzystym cyklem - co możemy wyeliminować ponieważ
cykl nie jest 3-regularny, lub $G = K_4$, gdzyż klika rozmiaru 4 jest 3-regularna (Ogólnie kliki rozmiaru $n$
$(n - 1)$-regularne) i tylko w tym jednym wypadku będziemy potrzebować 4 kolorów, ale nigdy więcej.
Zatem podsumowując odpowiedzi:
Zatem podsumowując odpowiedzi:
\begin{enumerate}[a)]
\item $\chi(X) \leq 1$ - \textbf{NIE}
\item $\chi(X) \leq 2$ - sprawdzenie dwudzielności - $O(n + m)$
\item $\chi(X) \leq 3$ - sprawdzenie czy $G \neq K_4$ - $O(1)$
\item $\chi(X) \leq 4$ - \textbf{TAK}
\item $\chi(X) \geq 1$ - \textbf{TAK} - zawsze potrzebny jest minimum 1 kolor
\item $\chi(X) \geq 2$ - \textbf{TAK} - minimalna liczba potrzebnych kolorów dla takiego grafu to 2
\item $\chi(X) \geq 3$ - jeżeli graf nie jest dwudzielny - to na pewno potrzebuje minimum 3 kolorów - $O(n+m)$
\item $\chi(X) \geq 4$ - graf potrzebuje 4 kolorów tylko, gdy jest kliką $K_4$ - $O(1)$.
\end{enumerate}
\begin{enumerate}[a)]
\item $\chi(X) \leq 1$ - \textbf{NIE}
\item $\chi(X) \leq 2$ - sprawdzenie dwudzielności - $O(n + m)$
\item $\chi(X) \leq 3$ - sprawdzenie czy $G \neq K_4$ - $O(1)$
\item $\chi(X) \leq 4$ - \textbf{TAK}
\item $\chi(X) \geq 1$ - \textbf{TAK} - zawsze potrzebny jest minimum 1 kolor
\item $\chi(X) \geq 2$ - \textbf{TAK} - minimalna liczba potrzebnych kolorów dla takiego grafu to 2
\item $\chi(X) \geq 3$ - jeżeli graf nie jest dwudzielny - to na pewno potrzebuje minimum 3 kolorów - $O(n+m)$
\item $\chi(X) \geq 4$ - graf potrzebuje 4 kolorów tylko, gdy jest kliką $K_4$ - $O(1)$.
\end{enumerate}
\end{solution}
\task{\NP-trudny problem \textit{Maximum Acyclic Subgraph} (\problem{MAS}) pyta o maksymalny zbiór łuków digrafu $D$,
@ -218,157 +218,157 @@ który generuje digraf acykliczny. Problem ten ma prosty algorym 2-aproksymacyjn
\tip{Jaki digraf definiuje górna/dolna macierz trójkątna macierzy $A$?}
\begin{solution}
Na wstępie - zadanie słabo opisuje problem i więcej informacji można wyciągnąć z nazwy problemu po angielsku niż jego opisu.
W problemie chodzi o znalezienie największego podgrafu acyklicznego w digrafie $D$.
\begin{figure}[H]
\begin{minipage}{.5\linewidth}%
\centering
\input{./gfx/mas-1.tex}
\end{minipage}%
\begin{minipage}{.5\linewidth}%
\begin{equation*}
A_D = \begin{bmatrix}
0 & 1 & 0 & 0 & 1 \\
0 & 0 & 1 & 0 & 0 \\
1 & 0 & 0 & 0 & 1 \\
0 & 0 & 1 & 0 & 0 \\
0 & 0 & 0 & 1 & 0 \\
\end{bmatrix}
\end{equation*}
\end{minipage}%
\caption{Przykładowy digraf $D$ wraz z maksymalnym podgrafem acyklicznym oraz macierzą sąsiedztwa.}
\end{figure}
Na wstępie - zadanie słabo opisuje problem i więcej informacji można wyciągnąć z nazwy problemu po angielsku niż jego opisu.
W problemie chodzi o znalezienie największego podgrafu acyklicznego w digrafie $D$.
\begin{figure}[H]
\begin{minipage}{.5\linewidth}%
\centering
\input{./gfx/mas-1.tex}
\end{minipage}%
\begin{minipage}{.5\linewidth}%
\begin{equation*}
A_D = \begin{bmatrix}
0 & 1 & 0 & 0 & 1 \\
0 & 0 & 1 & 0 & 0 \\
1 & 0 & 0 & 0 & 1 \\
0 & 0 & 1 & 0 & 0 \\
0 & 0 & 0 & 1 & 0 \\
\end{bmatrix}
\end{equation*}
\end{minipage}%
\caption{Przykładowy digraf $D$ wraz z maksymalnym podgrafem acyklicznym oraz macierzą sąsiedztwa.}
\end{figure}
Z zadania wiemy, że algorytm dzieli digraf $D$ na dwie części, na podstawie wskazówki \#1 możemy się domyślać, że ma to
coś wspólnego z macierzami trójkątnymi macierzy $A_D$. Zobaczmy, co te macierze reprezentują:
\begin{figure}[H]
\begin{minipage}{.5\linewidth}%
\centering
\input{./gfx/mas-2.tex}
\end{minipage}%
\begin{minipage}{.5\linewidth}%
\begin{equation*}
A_D = \begin{bmatrix}
0 & 1 & 0 & 0 & 1 \\
& 0 & 1 & 0 & 0 \\
& & 0 & 0 & 1 \\
& & & 0 & 0 \\
& & & & 0 \\
\end{bmatrix}
\end{equation*}
\end{minipage}%
\caption{Graf opisany przez macierz trójkątną górną macierzy $A_D$}
\end{figure}
Z zadania wiemy, że algorytm dzieli digraf $D$ na dwie części, na podstawie wskazówki \#1 możemy się domyślać, że ma to
coś wspólnego z macierzami trójkątnymi macierzy $A_D$. Zobaczmy, co te macierze reprezentują:
\begin{figure}[H]
\begin{minipage}{.5\linewidth}%
\centering
\input{./gfx/mas-2.tex}
\end{minipage}%
\begin{minipage}{.5\linewidth}%
\begin{equation*}
A_D = \begin{bmatrix}
0 & 1 & 0 & 0 & 1 \\
& 0 & 1 & 0 & 0 \\
& & 0 & 0 & 1 \\
& & & 0 & 0 \\
& & & & 0 \\
\end{bmatrix}
\end{equation*}
\end{minipage}%
\caption{Graf opisany przez macierz trójkątną górną macierzy $A_D$}
\end{figure}
\begin{figure}[H]
\begin{minipage}{.5\linewidth}%
\centering
\input{./gfx/mas-3.tex}
\end{minipage}%
\begin{minipage}{.5\linewidth}%
\begin{equation*}
A_D = \begin{bmatrix}
0 & & & & \\
0 & 0 & & & \\
1 & 0 & 0 & & \\
0 & 0 & 1 & 0 & \\
0 & 0 & 0 & 1 & 0 \\
\end{bmatrix}
\end{equation*}
\end{minipage}%
\caption{Graf opisany przez macierz trójkątną dolną macierzy $A_D$}
\end{figure}
\begin{figure}[H]
\begin{minipage}{.5\linewidth}%
\centering
\input{./gfx/mas-3.tex}
\end{minipage}%
\begin{minipage}{.5\linewidth}%
\begin{equation*}
A_D = \begin{bmatrix}
0 & & & & \\
0 & 0 & & & \\
1 & 0 & 0 & & \\
0 & 0 & 1 & 0 & \\
0 & 0 & 0 & 1 & 0 \\
\end{bmatrix}
\end{equation*}
\end{minipage}%
\caption{Graf opisany przez macierz trójkątną dolną macierzy $A_D$}
\end{figure}
Jak widać górna macierz trójkątna opisuje tylko takie krawędzie (łuki) $(u_1, u_2)$ że $u_1 < u_2$, czyli nie może
wystąpić cykl ponieważ to wymagałoby powrotu do wierzchołka, który już rozpatrywaliśmy - a to jest niemożliwe w takiej
macierzy. W macierzy dolnej sytuacja jest analogiczna, z tym że zachodzi warunek $u_1 > u_2$.
Jak widać górna macierz trójkątna opisuje tylko takie krawędzie (łuki) $(u_1, u_2)$ że $u_1 < u_2$, czyli nie może
wystąpić cykl ponieważ to wymagałoby powrotu do wierzchołka, który już rozpatrywaliśmy - a to jest niemożliwe w takiej
macierzy. W macierzy dolnej sytuacja jest analogiczna, z tym że zachodzi warunek $u_1 > u_2$.
Rozwiązanie takie gwarantuje brak cyklu, jednak nie jest dokładne, zastanówmy się więc jaki może być najgorszy przypadek.
Biorąc pod uwagę, że bierzemy połowę grafu możemy być pewni, że algorytm nie pomyli się bardziej niż 2-krotnie, ponieważ
w najgorszym wypadku cały graf jest już acykliczny, a my bierzemy jego połowę. Nie mamy jednak pewności co do tego,
czy faktycznie może istnieć acykliczny graf dokładnie 2 krotnie większy od swojej połówki i prawdopodobnie na tym można
skończyć dowód, jednak udowodnijmy że faktycznie istnieje taki przypadek w którym algorytm zwróci dokładnie
2-przybliżony.
Rozwiązanie takie gwarantuje brak cyklu, jednak nie jest dokładne, zastanówmy się więc jaki może być najgorszy przypadek.
Biorąc pod uwagę, że bierzemy połowę grafu możemy być pewni, że algorytm nie pomyli się bardziej niż 2-krotnie, ponieważ
w najgorszym wypadku cały graf jest już acykliczny, a my bierzemy jego połowę. Nie mamy jednak pewności co do tego,
czy faktycznie może istnieć acykliczny graf dokładnie 2 krotnie większy od swojej połówki i prawdopodobnie na tym można
skończyć dowód, jednak udowodnijmy że faktycznie istnieje taki przypadek w którym algorytm zwróci dokładnie
2-przybliżony.
Aby uzyskać taki graf w dolnej i górnej macierzy trójkątnej musimy mieć dokładnie taką samą ilość krawędzi, a ponadto
graf musi pozostać acykliczny. Nie jesteśmy w stanie stworzyć grafu o 2 wierzchołkach, który spełniałby ten warunek,
ponieważ albo algorytm zwróci odpowiedź dokładną, albo graf będzie cykliczny tj. wierzchołek 1 będzie połączony z 2 oraz
2 z 1. Jesteśmy w stanie skonstruować jednak taki graf 3 wierzchołkowy, ponieważ wystarczy że będziemy mieli dokładnie
jedno połączenie wierzchołka ,,mniejszego'' z ,,większym'' oraz dokładnie jedno ,,większego'' z ,,mniejszym'' przy czym jedynym
ograniczeniem jest to, że nie mogą to być te same wierzchołki.
Aby uzyskać taki graf w dolnej i górnej macierzy trójkątnej musimy mieć dokładnie taką samą ilość krawędzi, a ponadto
graf musi pozostać acykliczny. Nie jesteśmy w stanie stworzyć grafu o 2 wierzchołkach, który spełniałby ten warunek,
ponieważ albo algorytm zwróci odpowiedź dokładną, albo graf będzie cykliczny tj. wierzchołek 1 będzie połączony z 2 oraz
2 z 1. Jesteśmy w stanie skonstruować jednak taki graf 3 wierzchołkowy, ponieważ wystarczy że będziemy mieli dokładnie
jedno połączenie wierzchołka ,,mniejszego'' z ,,większym'' oraz dokładnie jedno ,,większego'' z ,,mniejszym'' przy czym jedynym
ograniczeniem jest to, że nie mogą to być te same wierzchołki.
\begin{figure}[H]
\begin{minipage}{.5\linewidth}%
\centering
\input{./gfx/mas-4.tex}
\end{minipage}%
\begin{minipage}{.5\linewidth}%
\begin{equation*}
A_D = \begin{bmatrix}
0 & 1 & 0 \\
0 & 0 & 0 \\
0 & 1 & 0 \\
\end{bmatrix}
\end{equation*}
\end{minipage}%
\caption{Graf opisany przez macierz trójkątną dolną macierzy $A_D$}
\end{figure}
\begin{figure}[H]
\begin{minipage}{.5\linewidth}%
\centering
\input{./gfx/mas-4.tex}
\end{minipage}%
\begin{minipage}{.5\linewidth}%
\begin{equation*}
A_D = \begin{bmatrix}
0 & 1 & 0 \\
0 & 0 & 0 \\
0 & 1 & 0 \\
\end{bmatrix}
\end{equation*}
\end{minipage}%
\caption{Graf opisany przez macierz trójkątną dolną macierzy $A_D$}
\end{figure}
W tym wypadku digraf $D$ jest acykliczny, jednak algorytm będzie w stanie znaleźć tylko podgraf $\{ (1, 2) \}$
lub $\{ (1, 3) \}$, czyli zwróci 2-krotnie za mały wynik.
W tym wypadku digraf $D$ jest acykliczny, jednak algorytm będzie w stanie znaleźć tylko podgraf $\{ (1, 2) \}$
lub $\{ (1, 3) \}$, czyli zwróci 2-krotnie za mały wynik.
Istnienie schematu (F)PTAS mówi nam, że jesteśmy w stanie znaleźć wielomianowy algorytm o dowolnie dobrym przybliżeniu,
tj. algorytm $(1 + \varepsilon)$-aproksymacyjny, gdzie $\varepsilon > 0$. Znalezienie takiego schematu jest trudne i jak
mniemam wykraczające poza obszar przedmiotu a na pewno kolokwium, jednak stosunkowo prosto udowodnić, że taki schemat
nie może istnieć. Wystarczy udowodnić, że dla każdych możliwych danych wejściowych istnieje taki epsilon, że z
odpowiedzi przybliżonej jesteśmy w stanie ze 100\% pewnością wywnioskować odpowiedź dokładną - a to znaczyłoby, że
posiadamy wielomianowy algorytm rozwiązujący dany problem \NP-trudny dokładnie, co znaczyłoby, że $\P = \NP$
Istnienie schematu (F)PTAS mówi nam, że jesteśmy w stanie znaleźć wielomianowy algorytm o dowolnie dobrym przybliżeniu,
tj. algorytm $(1 + \varepsilon)$-aproksymacyjny, gdzie $\varepsilon > 0$. Znalezienie takiego schematu jest trudne i jak
mniemam wykraczające poza obszar przedmiotu a na pewno kolokwium, jednak stosunkowo prosto udowodnić, że taki schemat
nie może istnieć. Wystarczy udowodnić, że dla każdych możliwych danych wejściowych istnieje taki epsilon, że z
odpowiedzi przybliżonej jesteśmy w stanie ze 100\% pewnością wywnioskować odpowiedź dokładną - a to znaczyłoby, że
posiadamy wielomianowy algorytm rozwiązujący dany problem \NP-trudny dokładnie, co znaczyłoby, że $\P = \NP$
W tym wypadku rozważamy problem maksymalizacyjny, czyli wiemy że odpowiedź dokładna $N_\text{opt}$ jest większa bądź
równa od odpowiedzi naszego algorytmu $N$:
\begin{equation}
N_{\text{opt}} \geq N
\end{equation}
W tym wypadku rozważamy problem maksymalizacyjny, czyli wiemy że odpowiedź dokładna $N_\text{opt}$ jest większa bądź
równa od odpowiedzi naszego algorytmu $N$:
\begin{equation}
N_{\text{opt}} \geq N
\end{equation}
Co za tym idzie, z definicji przybliżalności wynika, że:
\begin{equation*}
\frac{N_{\text{opt}}}{N} \leq 1+\varepsilon \iff N_\text{opt} \leq N(1 + \varepsilon)
\end{equation*}
Co za tym idzie, z definicji przybliżalności wynika, że:
\begin{equation*}
\frac{N_{\text{opt}}}{N} \leq 1+\varepsilon \iff N_\text{opt} \leq N(1 + \varepsilon)
\end{equation*}
łącząc te dwie wiadomości możemy wywnioskować, że
\begin{equation}
\label{eqn:2013:fptas}
N \leq N_\text{opt} \leq N + N\varepsilon
\end{equation}
łącząc te dwie wiadomości możemy wywnioskować, że
\begin{equation}
\label{eqn:2013:fptas}
N \leq N_\text{opt} \leq N + N\varepsilon
\end{equation}
ponieważ wiemy, że liczba krawędzi w grafie jest zawsze liczbą naturalną ($N_\text{opt} \in \mathbb{N}$ oraz $N \in \mathbb{N}$),
wystarczy że w nierówności \ref{eqn:2013:fptas} będzie spełniony warunek
ponieważ wiemy, że liczba krawędzi w grafie jest zawsze liczbą naturalną ($N_\text{opt} \in \mathbb{N}$ oraz $N \in \mathbb{N}$),
wystarczy że w nierówności \ref{eqn:2013:fptas} będzie spełniony warunek
\begin{equation}
\label{eqn:2013:fptas-condition}
N\varepsilon < 1
\end{equation}
\begin{equation}
\label{eqn:2013:fptas-condition}
N\varepsilon < 1
\end{equation}
A uzyskamy
\begin{equation*}
N \leq N_\text{opt} \leq N + N\varepsilon \Rightarrow N \leq N_\text{opt} \leq N
\end{equation*}
\begin{equation*}
N_\text{opt} = N
\end{equation*}
czyli algorytm zwróci nam odpowiedź dokładną.
A uzyskamy
Należy więc udowodnić że dla każdych danych wejściowych jesteśmy w stanie znaleźć dostatecznie mały $\varepsilon$,
który spowoduje że nasza odpowiedź będzie dokładna. Aby tego dokonać ograniczmy przybliżoną odpowiedź algorytmu z
dołu. Oczywistym jest, że w grafie o $m$ krawędziach nie będzie grafu o ilości krawędzi $N$ większej niż $m$, a już
na pewno większej niż $m+1$:
\begin{equation*}
N < m + 1
\end{equation*}
\begin{equation*}
N \leq N_\text{opt} \leq N + N\varepsilon \Rightarrow N \leq N_\text{opt} \leq N
\end{equation*}
\begin{equation*}
N_\text{opt} = N
\end{equation*}
Jeżeli teraz przedzielimy obie strony nierówności przez $m + 1$ uzyskamy po prawej stronie 1, dokładnie tak jak w nierówności \ref{eqn:2013:fptas-condition}
\begin{equation*}
\frac{1}{m+1}N < 1
\end{equation*}
czyli algorytm zwróci nam odpowiedź dokładną.
Należy więc udowodnić że dla każdych danych wejściowych jesteśmy w stanie znaleźć dostatecznie mały $\varepsilon$, który spowoduje że nasza odpowiedź będzie dokładna. Aby tego dokonać ograniczmy przybliżoną odpowiedź algorytmu z dołu. Oczywistym jest, że w grafie o $m$ krawędziach nie będzie grafu o ilości krawędzi $N$ większej niż $m$, a już na pewno większej niż $m+1$:
\begin{equation*}
N < m + 1
\end{equation*}
Jeżeli teraz przedzielimy obie strony nierówności przez $m + 1$ uzyskamy po prawej stronie 1, dokładnie tak jak w nierówności \ref{eqn:2013:fptas-condition}
\begin{equation*}
\frac{1}{m+1}N < 1
\end{equation*}
a co za tym idzie wystarczy, że $\varepsilon < \frac{1}{m+1}$ a będziemy w stanie wywnioskować dokładny wynik w czasie wielomianowym, co jest możliwe tylko gdy $\P = \NP$.
a co za tym idzie wystarczy, że $\varepsilon < \frac{1}{m+1}$ a będziemy w stanie wywnioskować dokładny wynik w czasie wielomianowym, co jest możliwe tylko gdy $\P = \NP$.
\end{solution}

View File

@ -1,4 +1,4 @@
%! TEX root = main.tex
%! TEX root = ../main.tex
\section{25.01.2018 kolokwium \#2}
\task Rozważ problem wyznaczania wartości indeksu chromatycznego $n$-wierzchołkowego grafu $G$, czyli $\chi'(G)$.
@ -128,15 +128,48 @@ Uzasadnij prawdziwość lub fałszywość następujących twierdzeń dotyczącyc
\begin{table}[H]
\centering
\begin{tabular}{rl|c|c}
& klasa & Weryfikowalne & Rozwiązywalne \\\hline
& klasa & Weryfikowalne \dag & Rozwiązywalne \dag \\\hline
\subtask & \P & & \\
\subtask & \NP & & \\
\subtask & \NPC & & \\
\subtask & \NP-trudne & & \\
\end{tabular}
\end{table}
\noindent\dag\ w czasie wielomianowym
\note{W niektórych wypadkach więcej niż jedna odpowiedź jest poprawna}
\begin{solution}
Z definicji, problemy klasy \P da się rozwiązać w czasie wielomianowym - a ponieważ da się je rozwiązać to również
da sie sprawdzić poprawnosć rozwiązania (wystarczy rozwiązać i porównać) w czasie wielomianowym.
Wiadomo, że wszystkie problemy z klasy \NP są weryfikowalne w czasie wielomianowym - nie wiemy natomiast czy
problemy te możemy w czasie wielomianowym rozwiązać. Jednak należy pamiętać, że klasa $\P \subseteq \NP$ - zatem w
klasie \NP z pewnością są też problemy, które możemy rozwiązać w czasie wielomianowym.
Ponieważ $\NPC \subseteq \NP$ to na pewno ich rozwiązania da się zweryfikować w czasie wielomianowym. Gdybyśmy wiedzieli
jednak, że jakiegoś problemu \NPC nie da się rozwiązać w czasie wielomianowym to znaczyłoby to, że $\P \neq \NP$,
podobnie gdybyśmy wiedzieli, że któryś problem \NPC da się rozwiązać w czasie wielomianowym to na pewno prawdą
byłoby, że $\P = \NP$. A że nie wiemy czy $\P = \NP$ tak też nie wiemy nic na temat rozwiązywalności problemów \NPC
w czasie wielomianowym.
Problemy \NP-trudne to problemy conajmniej tak trudne jak najtrudniejsze problemy w klasie \NP, zatem w pewnym
sensie prawdą jest, że $\NPC = \NP \cap \NPH$. Stąd wynika, że w klasie \NPH na pewno są problemy, które da się
zweryfikować w czasie wielomianowym, o których rozwiązaniu nic nie wiadomo. Ponieważ jednak są to propblemy
\textit{conajmniej} tak trudne to klasa ta zawiera też problemy niealgorytmiczne, których nie da się ani rozwiązać
ani zweryfikować w czasie rzeczywistym.
\begin{table}[H]
\centering
\begin{tabular}{rl|c|c}
& klasa & Weryfikowalne \dag & Rozwiązywalne \dag \\\hline
\subtask & \P & \textbf{TAK} & \textbf{TAK} \\
\subtask & \NP & \textbf{TAK} & \textbf{TAK}, \textbf{NW} \\
\subtask & \NPC & \textbf{TAK} & \textbf{NW} \\
\subtask & \NP-trudne & \textbf{TAK}, \textbf{NIE} & \textbf{NW}, \textbf{NIE} \\
\end{tabular}
\end{table}
\end{solution}
\task Algorytm \textit{Largest First} (\problem{LF}) dla kolorowania wierzchołków grafu maluje je zachłannie
poczynając od wierzchołka o najwyższym stopniu i kończąc na weierzchołku o najniższym stopniu.
\subtask Oszacuj złożoność obliczeniową algorytmu \problem{LF} jako funnkcję $n$ za pomocą symbolu $\Theta$
@ -146,11 +179,10 @@ poczynając od wierzchołka o najwyższym stopniu i kończąc na weierzchołku o
\subtask Udowodnij, że \problem{LF} jest $k$-bezwzględnie aproksymacyjny i $l$-wzzględnie aproksymacyjny w odniesieniu
do grafów kubicznych tj. ustal wartości k i l
\task \textit{Kolorowanie Kosztowe} (\problem{KK}) polega na tym, że kolory mają swoje koszty
$c_1 \leq c_2 \leq \ldots \leq c_n$ i za każdym razem, gdy kolorujemy kolejny wierzchołek, przydzielamy mu koszt tego
koloru. Problem polaga na tym, by tak pokolrowoać graf aby sumaryczny koszt kolorowania $\text{skk}(G)$ wszystkich
wierzchołków był jak najmniejszy. Udowodnij, że nawet jeżeli $c_1 = c_2 = \ldots = c_k \leq c_{k+1}$ to problem ten
jest \NP-trudny.
\task \textit{Kolorowanie Kosztowe} (\problem{KK}) polega na tym, że kolory mają swoje koszty $c_1 \leq c_2 \leq \ldots
\leq c_n$ i za każdym razem, gdy kolorujemy kolejny wierzchołek, przydzielamy mu koszt tego koloru. Problem polaga na
tym, by tak pokolrowoać graf aby sumaryczny koszt kolorowania $\text{skk}(G)$ wszystkich wierzchołków był jak
najmniejszy. Udowodnij, że nawet jeżeli $c_1 = c_2 = \ldots = c_k \leq c_{k+1}$ to problem ten jest \NP-trudny.
\begin{solution}
Aby udowodnić, że problem jest \NP-trudny musimy udowodnić, że jego wersja decyzyjna jest \NPC. W wypadku tego
@ -177,14 +209,14 @@ jest \NP-trudny.
poniesiemy będzie na pewno większy niż $n$. Zatem graf da się pokolorować kosztem $n$ tylko kiedy graf da się
pokolorować $k$ kolorami, gdzie koszt każdego koloru wynosi $1$. Nasza $\alpha$-redukcja będzie zatem następująca:
\begin{equation*}
G, k \rightarrow \underset{G}{G}, \underset{c}{n}, \underset{c_k}{1}, \underset{k}{k}, \underset{c_{k+1}}{2}, \underset{c_{k+1}}{2}, \ldots
G, k \rightarrow \underset{G}{G}, \underset{c}{n}, \underset{c_k}{1}, \underset{k}{k}, \underset{c_{k+1}}{2}, \underset{c_{k+2}}{2}, \ldots
\end{equation*}
\end{solution}
\task Alicja jest częstym klientem sklepu \textit{SuperShop} i uzbierała 5 kuponów rabatowych o różnych wartościach
zniżek. Ma zapisane na liście zakupowej $n$ produktów, które chce dzisiaj kupić oraz ich ceny. Chce pójść dzisiaj do
zniżek. Ma zapisane na liście zakupowej $n$ produktów, które chce dzisiaj kupić oraz ich ceny. Chce pójść dzisiaj do
sklepu bez gotówki i kupić wybrane produkty używajac tylko kuponów. Nie chce natomiast zmarnować żadnego kuponu, czyli
używa tylko tych, których wartość zostanie w pełni wykorzystana. Zastnawia się czy istnieje zestaw produktów i
używa tylko tych, których wartość zostanie w pełni wykorzystana. Zastnawia się czy istnieje zestaw produktów i
odpowiednia kombinacja kuponów, by mogła pójść na zakupy i kupić cokolwiek?
\note{Wszystkie kwoty są zaokrąglane do pełnych złotówek}